Spinor Representation of Lorentz Transformations: Solving the Puzzle

In summary: Yes, I am using the relation \omega_{ij}\gamma^{i}\gamma^{j}=\frac{1}{2}\omega_{ij}\left[\gamma^{i},\,\gamma^{j}\right] . And yes, we choose \theta^{k} = - \frac{1}{2}\epsilon^{ijk}\omega_{ij} for convenience.
  • #1
Frank Castle
580
23
I've been working my way through Peskin and Schroeder and am currently on the sub-section about how spinors transform under Lorentz transformation. As I understand it, under a Lorentz transformation, a spinor ##\psi## transforms as $$\psi\rightarrow S(\Lambda)\psi$$ where $$S(\Lambda)=\exp\left(-\frac{i}{2}\omega_{\mu\nu}\Sigma^{\mu\nu}\right)$$ with $$\Sigma^{\mu\nu}=\frac{i}{4}\left[\gamma^{\mu},\,\gamma^{\nu}\right]=-\Sigma^{\nu\mu}$$ Then, in the Weyl representation we have that $$\Sigma^{0i}=-\frac{i}{2}\left(\begin{matrix}\sigma^{i}&&0\\ 0&&-\sigma^{i}\end{matrix}\right)$$ and $$\Sigma^{ij}=\frac{i}{2}\varepsilon^{ijk}\left(\begin{matrix}\sigma^{k}&&0\\ 0&&\sigma^{k}\end{matrix}\right)$$ Given this, what confuses me is how one ends up with the following left-handed and right-handed transformations: $$S(\Lambda)_{L}=\exp\left(-\frac{\mathbf{\beta}\cdot\mathbf{\sigma}}{2}+i\frac{\mathbf{\theta}\cdot\mathbf{\sigma}}{2}\right) \\ \\ S(\Lambda)_{R}=\exp\left(\frac{\mathbf{\beta}\cdot\mathbf{\sigma}}{2}+i\frac{\mathbf{\theta}\cdot\mathbf{\sigma}}{2}\right)$$ Where does the additional ##i## come from in the spatial rotations term?

I have read from other sources, that the parameters ##\omega_{\mu\nu}## are defined such that ##\omega_{0i}=\beta_{i}## and ##\omega_{ij}=\varepsilon_{ijk}\theta^{k}##, which are the boost and rotation parameters respectively. Given these, however, I can't arrive at the above expressions. For example, for ##S(\Lambda)_{L}## I obtain
$$S(\Lambda)_{L}=\exp\left(-\frac{\mathbf{\beta}\cdot\mathbf{\sigma}}{4}+\varepsilon_{ijk}\varepsilon^{ijl}\frac{\theta^{k}\sigma^{l}}{4}\right)=\exp\left(-\frac{\mathbf{\beta}\cdot\mathbf{\sigma}}{4}+\frac{\mathbf{\theta}\cdot\mathbf{\sigma}}{2}\right)$$ where I have used that ##\varepsilon_{ijk}\varepsilon^{ijl}=2\delta_{kl}##.

Would someone be able to explain this to me as I'm really stuck on this point at the moment.
 
Physics news on Phys.org
  • #2
The rotations are represented by the usual SU(2) operations on the left and right-handed parts of the Dirac operator. Thus, there's a factor ##\mathrm{i}## too much in your definition of ##\Sigma^{ij}##!
 
  • #3
Frank Castle said:
Where does the additional ##i## come from in the spatial rotations term?
Would someone be able to explain this to me as I'm really stuck on this point at the moment.

Using the antisymmetry of the Lorentz parameters, you can write [tex]-\frac{i}{2}\omega_{\mu\nu}\Sigma^{\mu\nu} = \frac{1}{4} \omega_{\mu\nu} \gamma^{\mu}\gamma^{\nu} .[/tex] Expanding the summation leads to [tex]-\frac{i}{2}\omega_{\mu\nu}\Sigma^{\mu\nu} = \frac{1}{4}\omega_{0k}\gamma^{0}\gamma^{k} + \frac{1}{4}\omega_{k0}\gamma^{k}\gamma^{0} + \frac{1}{4}\omega_{ij}\gamma^{i}\gamma^{j} .[/tex] The first two terms are equal, because [itex]\omega_{k0}=-\omega_{0k}[/itex] and [itex]\gamma^{k}\gamma^{0} = - \gamma^{0}\gamma^{k}[/itex]. Thus

[tex]-\frac{i}{2}\omega_{\mu\nu}\Sigma^{\mu\nu} = \frac{1}{2} \omega_{0k}\gamma^{0}\gamma^{k} + \frac{1}{4} \omega_{ij}\gamma^{i}\gamma^{j} . \ \ \ (1)[/tex]

Now, in the chiral representation we have

[tex]\gamma^{0}\gamma^{k} = \begin{pmatrix} 0 & 1 \\ 1 & 0 \end{pmatrix} \begin{pmatrix} 0 & \sigma^{k} \\ - \sigma^{k} & 0 \end{pmatrix} = \begin{pmatrix} - \sigma^{k} & 0 \\ 0 & \sigma^{k} \end{pmatrix} ,[/tex]

[tex]\gamma^{i}\gamma^{j} = \begin{pmatrix} 0 & \sigma^{i} \\ - \sigma^{i} & 0 \end{pmatrix} \begin{pmatrix} 0 & \sigma^{j} \\ - \sigma^{j} & 0 \end{pmatrix} = - i \epsilon^{ijk} \begin{pmatrix} \sigma^{k} & 0 \\ 0 & \sigma^{k} \end{pmatrix} .[/tex]

Substituting these expressions in (1), you get

[tex]-\frac{i}{2}\omega_{\mu\nu}\Sigma^{\mu\nu} = \omega_{0k} \begin{pmatrix} - \frac{\sigma^{k}}{2} & 0 \\ 0 & \frac{\sigma^{k}}{2} \end{pmatrix} + i \left( - \frac{1}{2}\epsilon^{ijk}\omega_{ij} \right) \begin{pmatrix} \frac{\sigma^{k}}{2} & 0 \\ 0 & \frac{\sigma^{k}}{2} \end{pmatrix} .[/tex]

Now, if you define the parameters [itex]\beta^{k} = \omega_{0k}[/itex] and [itex]\theta^{k} = - \frac{1}{2}\epsilon^{ijk}\omega_{ij}[/itex], you get

[tex]-\frac{i}{2}\omega_{\mu\nu}\Sigma^{\mu\nu} = \begin{pmatrix} \frac{1}{2}\left( - \vec{\beta} + i \vec{\theta} \right) \cdot \vec{\sigma} & 0 \\ 0 & \frac{1}{2}\left( \vec{\beta} + i \vec{\theta} \right) \cdot \vec{\sigma} \end{pmatrix} .[/tex]
 
  • Like
Likes vanhees71
  • #4
samalkhaiat said:
Using the antisymmetry of the Lorentz parameters, you can write [tex]-\frac{i}{2}\omega_{\mu\nu}\Sigma^{\mu\nu} = \frac{1}{4} \omega_{\mu\nu} \gamma^{\mu}\gamma^{\nu} .[/tex] Expanding the summation leads to [tex]-\frac{i}{2}\omega_{\mu\nu}\Sigma^{\mu\nu} = \frac{1}{4}\omega_{0k}\gamma^{0}\gamma^{k} + \frac{1}{4}\omega_{k0}\gamma^{k}\gamma^{0} + \frac{1}{4}\omega_{ij}\gamma^{i}\gamma^{j} .[/tex] The first two terms are equal, because [itex]\omega_{k0}=-\omega_{0k}[/itex] and [itex]\gamma^{k}\gamma^{0} = - \gamma^{0}\gamma^{k}[/itex]. Thus

[tex]-\frac{i}{2}\omega_{\mu\nu}\Sigma^{\mu\nu} = \frac{1}{2} \omega_{0k}\gamma^{0}\gamma^{k} + \frac{1}{4} \omega_{ij}\gamma^{i}\gamma^{j} . \ \ \ (1)[/tex]

Now, in the chiral representation we have

[tex]\gamma^{0}\gamma^{k} = \begin{pmatrix} 0 & 1 \\ 1 & 0 \end{pmatrix} \begin{pmatrix} 0 & \sigma^{k} \\ - \sigma^{k} & 0 \end{pmatrix} = \begin{pmatrix} - \sigma^{k} & 0 \\ 0 & \sigma^{k} \end{pmatrix} ,[/tex]

[tex]\gamma^{i}\gamma^{j} = \begin{pmatrix} 0 & \sigma^{i} \\ - \sigma^{i} & 0 \end{pmatrix} \begin{pmatrix} 0 & \sigma^{j} \\ - \sigma^{j} & 0 \end{pmatrix} = - i \epsilon^{ijk} \begin{pmatrix} \sigma^{k} & 0 \\ 0 & \sigma^{k} \end{pmatrix} .[/tex]

Substituting these expressions in (1), you get

[tex]-\frac{i}{2}\omega_{\mu\nu}\Sigma^{\mu\nu} = \omega_{0k} \begin{pmatrix} - \frac{\sigma^{k}}{2} & 0 \\ 0 & \frac{\sigma^{k}}{2} \end{pmatrix} + i \left( - \frac{1}{2}\epsilon^{ijk}\omega_{ij} \right) \begin{pmatrix} \frac{\sigma^{k}}{2} & 0 \\ 0 & \frac{\sigma^{k}}{2} \end{pmatrix} .[/tex]

Now, if you define the parameters [itex]\beta^{k} = \omega_{0k}[/itex] and [itex]\theta^{k} = - \frac{1}{2}\epsilon^{ijk}\omega_{ij}[/itex], you get

[tex]-\frac{i}{2}\omega_{\mu\nu}\Sigma^{\mu\nu} = \begin{pmatrix} \frac{1}{2}\left( - \vec{\beta} + i \vec{\theta} \right) \cdot \vec{\sigma} & 0 \\ 0 & \frac{1}{2}\left( \vec{\beta} + i \vec{\theta} \right) \cdot \vec{\sigma} \end{pmatrix} .[/tex]

Great answer, thanks.
There's just a couple of things that I'm not quite sure about, how did you get [tex]\gamma^{i}\gamma^{j} = \begin{pmatrix} 0 & \sigma^{i} \\ - \sigma^{i} & 0 \end{pmatrix} \begin{pmatrix} 0 & \sigma^{j} \\ - \sigma^{j} & 0 \end{pmatrix} = - i \epsilon^{ijk} \begin{pmatrix} \sigma^{k} & 0 \\ 0 & \sigma^{k} \end{pmatrix} .[/tex] Naively, I get $$\gamma^{i}\gamma^{j} = \begin{pmatrix} 0 & \sigma^{i} \\ - \sigma^{i} & 0 \end{pmatrix} \begin{pmatrix} 0 & \sigma^{j} \\ - \sigma^{j} & 0 \end{pmatrix} = \begin{pmatrix} -\sigma^{i}\sigma^{j} & 0\\ 0 & -\sigma^{i}\sigma^{j} \end{pmatrix}$$ If I then use ##\left[\sigma^{i},\,\sigma^{j}\right]=2i\varepsilon^{ijk}\sigma^{k}## then I'll just end up with ##\sigma^{i}\sigma^{j}=\sigma^{j}\sigma^{i}+2i\varepsilon^{ijk}\sigma^{k}##?! Are you simply using that $$\omega_{ij}\gamma^{i}\gamma^{j}=\frac{1}{2}\omega_{ij}\left[\gamma^{i},\,\gamma^{j}\right]$$ such that in the Chiral representation $$\omega_{ij}\gamma^{i}\gamma^{j}=\omega_{ij}\begin{pmatrix} -\left[\sigma^{i},\,\sigma^{j}\right] & 0\\ 0 & -\left[\sigma^{i},\,\sigma^{j}\right] \end{pmatrix}=-2i\omega_{ij}\varepsilon^{ijk}\begin{pmatrix} \sigma^{k} & 0\\ 0 & \sigma^{k} \end{pmatrix}$$

Also, do we simple choose that [itex]\theta^{k} = - \frac{1}{2}\epsilon^{ijk}\omega_{ij}[/itex] for convenience (absorbing the minus sign)?
 
  • #5
vanhees71 said:
The rotations are represented by the usual SU(2) operations on the left and right-handed parts of the Dirac operator. Thus, there's a factor ##\mathrm{i}## too much in your definition of ##\Sigma^{ij}##!

Good point, I'd forgotten about the ##i## already in the definition of ##\Sigma^{\mu\nu}##!
Also, are the choices of for the forms of the parameters somewhat arbitrary? For example, does one choose ##\theta^{k}=-\frac{1}{2}\varepsilon^{ijk}\omega_{ij}## purely for convenience to absorb the extra factor of ##1/2## floating around?!
 
  • #6
Frank Castle said:
Naively, I get $$\gamma^{i}\gamma^{j} = \begin{pmatrix} 0 & \sigma^{i} \\ - \sigma^{i} & 0 \end{pmatrix} \begin{pmatrix} 0 & \sigma^{j} \\ - \sigma^{j} & 0 \end{pmatrix} = \begin{pmatrix} -\sigma^{i}\sigma^{j} & 0\\ 0 & -\sigma^{i}\sigma^{j} \end{pmatrix}$$
So, [tex]\omega_{ij}\gamma^{i}\gamma^{j} = - \begin{pmatrix} \omega_{ij}\sigma^{i}\sigma^{j} & 0 \\ 0 & \omega_{ij}\sigma^{i}\sigma^{j} \end{pmatrix} .[/tex]

Now, use the identity [itex]\sigma^{i}\sigma^{j} = \delta^{ij} I_{2} + i \epsilon^{ijk}\sigma^{k}[/itex] together with [itex]\omega_{ij}\delta^{ij} = 0[/itex].
Remember that this is a rotation in the [itex](ij)[/itex]-plane, i.e., [itex]i \neq j[/itex]. And, in this case, [itex]\sigma^{i}\sigma^{j} = i \epsilon^{ijk}\sigma^{k}[/itex].

Also, do we simple choose that [itex]\theta^{k} = - \frac{1}{2}\epsilon^{ijk}\omega_{ij}[/itex] for convenience?

Yes. You can choose any sign for [itex]\theta^{k}[/itex] (and for [itex]\beta^{k}[/itex]). I picked the minus sign so that my results agree with the expressions you wrote for [itex]S_{L}(\Lambda)[/itex] and [itex]S_{R}(\Lambda)[/itex].
 
  • #7
samalkhaiat said:
So, [tex]\omega_{ij}\gamma^{i}\gamma^{j} = - \begin{pmatrix} \omega_{ij}\sigma^{i}\sigma^{j} & 0 \\ 0 & \omega_{ij}\sigma^{i}\sigma^{j} \end{pmatrix} .[/tex]

Now, use the identity [itex]\sigma^{i}\sigma^{j} = \delta^{ij} I_{2} + i \epsilon^{ijk}\sigma^{k}[/itex] together with [itex]\omega_{ij}\delta^{ij} = 0[/itex].
Remember that this is a rotation in the [itex](ij)[/itex]-plane, i.e., [itex]i \neq j[/itex]. And, in this case, [itex]\sigma^{i}\sigma^{j} = i \epsilon^{ijk}\sigma^{k}[/itex].

Yes. You can choose any sign for [itex]\theta^{k}[/itex] (and for [itex]\beta^{k}[/itex]). I picked the minus sign so that my results agree with the expressions you wrote for [itex]S_{L}(\Lambda)[/itex] and [itex]S_{R}(\Lambda)[/itex].

Ok great, thanks for your help. I assume what I wrote at the end of post #4 is correct?!
 

1. What is a spinor representation?

A spinor representation is a mathematical representation of a physical system that involves spin, which is an intrinsic property of particles. It is used to describe the behavior of particles under rotations and Lorentz transformations in special relativity.

2. What is the significance of the spinor representation in solving the puzzle of Lorentz transformations?

The spinor representation allows for a better understanding of how particles behave under Lorentz transformations, which are essential in special relativity. It helps to resolve the puzzle of how particles with half-integer spin behave differently than those with integer spin.

3. How is the spinor representation related to the Lorentz group?

The spinor representation is a mathematical representation of the Lorentz group, which is a mathematical group that describes the symmetries of space and time in special relativity. It is used to describe how particles behave under these symmetries.

4. What are some applications of the spinor representation in physics?

The spinor representation has wide applications in different areas of physics, including quantum mechanics, particle physics, and general relativity. It is used to describe the behavior of particles with spin, such as electrons, quarks, and neutrinos, and is also used in the study of black holes and other astrophysical phenomena.

5. Is the spinor representation the only way to solve the puzzle of Lorentz transformations?

No, there are other mathematical approaches to solving the puzzle of Lorentz transformations, such as using tensor representations or geometric algebra. However, the spinor representation is a particularly useful and elegant way to understand the behavior of particles under Lorentz transformations in special relativity.

Similar threads

  • Special and General Relativity
Replies
1
Views
783
  • Special and General Relativity
Replies
6
Views
1K
  • Special and General Relativity
Replies
14
Views
2K
  • Special and General Relativity
Replies
7
Views
5K
  • Special and General Relativity
Replies
5
Views
2K
Replies
2
Views
768
  • Special and General Relativity
Replies
3
Views
988
  • Special and General Relativity
Replies
9
Views
1K
  • Special and General Relativity
Replies
34
Views
4K
  • Special and General Relativity
Replies
9
Views
1K
Back
Top